The intensity of light is reduced by the coating for λ=430nm as compared to no coating by a factor of 0.316.
The reflectivity of the coating is dependent on the wavelength of light, and the reflectivity for λ=430nm is greater than the reflectivity for λ=540nm.
The reflectivity of the coating can be calculated using the Fresnel equations, which describe the reflection and transmission of light at an interface between two different media. The reflectivity R for normal incidence is given by:
R = ((n₁ - n₂)/(n₁ + n₂))²
Where n₁ and n₂ are the indices of refraction of the two media.
For λ=540nm, the reflectivity is zero, which means that n₁ = n₂.
For λ=430nm, the reflectivity is greater than zero, which means that n₁ ≠ n₂.
The factor by which the intensity is reduced can be calculated by taking the ratio of the reflectivities for the two wavelengths:
factor = R(λ=430nm)/R(λ=540nm)
Since R(λ=540nm) = 0, the factor is simply equal to R(λ=430nm).
Using the Fresnel equations and the indices of refraction for the two media, we can calculate the reflectivity for λ=430nm and find the factor by which the intensity is reduced:
R(λ=430nm) = ((n₁ - n₂)/(n₁ + n₂))² = 0.316
Hence, the intensity of light is reduced by a factor of 0.316.
Learn more about Fresnel equations here: https://brainly.com/question/28197248.
#SPJ11
Which one(s) of the following statements is (are) correct for elastic collision between two objects?
(A) Total momentum is not conserved, but total kinetic energy is conserved.
(B) Neither the total momentum nor the total kinetic energy is conserved.
(C) Bother the total momentum and the total kinetic energy are conserved.
(D) The amount of kinetic energy lost by one object is the same as the amount of kinetic energy gained by the other object.
(E) The magnitude of the momentum lost by one object is the same as the magnitude of the momentum gained by the other object.
(F) Total momentum is conserved, but total kinetic energy is not conserved.
Option (C) is correct for an elastic collision between two objects.
i.e. both the total momentum and total kinetic energy are conserved.
The sum of the momentum of the two objects before the collision is equal to the sum of the momentum of the two objects after the collision, and the sum of their kinetic energy remains the same.In an elastic collision, both the total momentum and total kinetic energy are conserved.This is different from an inelastic collision where some of the kinetic energy is converted into other forms of energy and lost.
Option (A) is incorrect because the total momentum is conserved, not lost.
Option (B) is incorrect because both momentum and kinetic energy are conserved.
Option (C) is correct for an elastic collision between two objects. In an elastic collision, both the total momentum and total kinetic energy are conserved.
Option (D) is partially correct because the kinetic energy lost by one object is gained by the other object, but it doesn't take into account the conservation of the total kinetic energy.
Option (E) is partially correct because the magnitude of momentum lost by one object is gained by the other object, but it doesn't take into account the conservation of total momentum.
Option (F) is incorrect because total momentum is conserved in elastic collisions.
Learn more about elastic collision here,
https://brainly.com/question/31318883
#SPJ11
The correct statement for elastic collision between two objects is (C) Both the total momentum and the total kinetic energy are conserved.
In an elastic collision, the two objects collide and rebound without any energy loss due to deformation or heat transfer. The kinetic energy is conserved because the total initial kinetic energy is equal to the total final kinetic energy after the collision. Additionally, the total momentum of the system is also conserved because the momentum of one object is transferred to the other during the collision. Therefore, the momentum gained by one object is equal to the momentum lost by the other object, making statement (E) also correct. Statement (A) is incorrect because the total momentum is also conserved. Statement (B) is incorrect because at least one of the conservation laws is always true in an elastic collision. Statement (D) is incorrect because the kinetic energy gained and lost by the objects is not necessarily the same. Statement (F) is also incorrect because the total momentum is conserved in elastic collisions.
learn more about elastic collision Refer: https://brainly.com/question/12644900
#SPJ11
The type of energy that depends on position is called
a.
potential energy.
b.
electrical energy.
c.
magnetic energy.
d.
solar energy.
Answer:
a. potential energy
Explanation:
ur welcome
If the electron is continuing in a horizontal straight line, express the magnitude of the magnetic field in terms of v and e.
If an electron is moving in a horizontal straight line, it means that there is no force acting on it in the horizontal direction. However,
if there is a magnetic field present, it will exert a force on the moving electron in a direction perpendicular to both the velocity of the electron and the magnetic field.
The magnitude of this force is given by the equation F = Bqv, where F is the force, B is the magnitude of the magnetic field, q is the charge of the electron, and v is the velocity of the electron.
Since we know that the electron is moving in a straight line, we can assume that the force acting on it is balanced by some other force, such as the electrostatic force.
Therefore, we can set the magnitude of the magnetic force equal to the magnitude of the electrostatic force and solve for B.
Assuming the electron has a charge of e, and the electrostatic force is given by F = eqE, where E is the electric field, we can set the two forces equal to each other and get:
Bqv = eqE
Simplifying this equation, we get:
B = E(v/e)
Therefore, the magnitude of the magnetic field in terms of v and e is given by B = E(v/e). This equation shows that the magnitude of the magnetic field is proportional to
the electric field and the velocity of the electron, and inversely proportional to the charge of the electron.
To know more about electron refer here
https://brainly.com/question/12001116#
#SPJ11
Which element on the periodic table fights dandruff, directs drivers, and makes copies of your homework?
Answer:
Talc is the known mineral
What geologic forces drive tectonic movements?
A- Earth's rotation, geothermal activity, gravitational forces
B- plate subduction, convergence and divergence
C- Earth's orbit, solar winds, electromagnetic waves
D- seismic movement, volcanic activity, plate tension
Seismic movement, Volcanic Activity, Plate Tension
If P and Q are two adjacent sides of a parallelogram. Then which of the following is incorrect, options are as follows:
Answer:
a) P+Q represents one side.
Explanation:
As they are adjacent side of a parallelogram P and Q will be 2 individual sides and hence will not represent one side.
So that statement is incorrect
HOPE IT HELPS YOU!!#IndianMurgaTwo astronauts are taking a spacewalk outside the International Space Station. The first astronaut has a mass of 85.2 kg. The second has a mass of 65.4 kg. Initially, both astronauts have zero velocity relative to each other. Then, the astronauts push against each other, giving the first astronaut a final velocity of 1.2 m/s to the left. If the momentum of the system is conserved, what is the final velocity of the second person?
A. 1.3 m/s to the left
B. 1.6 m/s to the right
C. 1.6 m/s to the left
D. 1.3 m/s to the right
The final velocity of the second person after the collision is determined as 1.6 m/s to the right.
Final velocity of the second person
The final velocity of the second person is determined by applying the principle of conservation of linear momentum.
m1u1 + m2u2 = m1v1 + m2v2
85.2(0) + 65.4(0) = 85.2(-1.2) + 65.4(v2)
0 = -102.24 + 65.4v2
65.4v2 = 102.24
v2 = 1.56 m/s to the right
Thus, the final velocity of the second person after the collision is determined as 1.6 m/s to the right.
Learn more about linear momentum here: https://brainly.com/question/7538238
#SPJ1
one cycle of the power dissipated by a resistor ( ) is given by this periodic signal repeats in both directions of time. what is the amplitude of the pwm voltage signal applied across the 600- resistor?
The amplitude of the PWM voltage signal applied across the 600-ohm resistor can be determined by first calculating the power dissipated and then using Ohm's Law and the power formula.
To find the amplitude of the PWM voltage signal, we need to know the power dissipated by the resistor during one cycle. However, the given information does not provide the power dissipated by the resistor or any other details of the periodic signal. If you can provide more information about the power dissipated, I would be happy to help you with the calculation.
In order to calculate the amplitude of the PWM voltage signal applied across the 600-ohm resistor, more information is needed about the power dissipation during one cycle. Please provide the necessary details, and I will be glad to assist you with your question.
To know more about Ohm's Law, visit:
https://brainly.com/question/1247379
#SPJ11
at what speed does an acoustic wave propagate in an incompressible flow?
In an incompressible flow, the speed at which an acoustic wave propagates depends on the physical properties of the medium through which it is traveling. Generally speaking, the speed of an acoustic wave in an incompressible flow is slower than in a compressible flow, since the latter can support pressure waves that move faster than sound.
The speed of an acoustic wave in an incompressible flow can be calculated using the following formula:
c = √(K/ρ)
where c is the speed of the wave, K is the bulk modulus of the medium (a measure of its resistance to compression), and ρ is its density.
For example, if we assume a bulk modulus of 2.3 GPa and a density of 1000 kg/m^3 (typical values for water), we get a speed of approximately 1500 m/s for an acoustic wave in an incompressible flow.
It's worth noting that this speed can vary depending on the exact conditions of the flow, as well as any obstructions or other features that might affect the propagation of the wave. Nonetheless, the above formula provides a useful starting point for understanding the speed of acoustic waves in incompressible flows.
To Know more about bulk modulus visit:
brainly.com/question/30505066
#SPJ11
Because the current surge in starting multiple motors is too great for the system, there must be a delay between the starting of each motor.
Because the current surge in starting multiple motors is too great for the system, a delay between the starting of each motor is necessary.
When multiple motors start simultaneously, they draw a significant amount of current, resulting in a high inrush current that can overload the electrical system. To prevent this, a delay is introduced between the starting of each motor. This delay allows the system to stabilize and accommodate the initial surge in current before the next motor is started. By staggering the motor start times, the overall current demand is distributed more evenly, reducing the strain on the electrical system. This practice helps to prevent voltage drops, voltage fluctuations, and potential damage to electrical components. Therefore, introducing a delay between the starting of each motor is essential to ensure the proper functioning and longevity of the system.
To learn more about current surge in starting multiple motors, Click here:
https://brainly.com/question/31061865
#SPJ11
What is the density of a piece of quartz with a mass of 30g and a volume of 6cm^3
Answer:
\(\boxed {\boxed {\sf d= 5 \ g/cm^3}}\)
Explanation:
Density can be found by dividing the mass by the volume.
\(d=\frac{m}{v}\)
The mass of the quartz is 30 grams and the volume is 6 cubic centimeters.
\(m=30 \ g \\v= 6 \ cm^3\)
Substitute the values into the formula.
\(d=\frac{30 \ g}{6 \ cm^3}\)
Divide.
\(d= 5 \ g/cm^3\)
The density of this piece of quartz is 5 grams per cubic centimeter.
What are the energy transformations that occur in an internal combustion engine?
Answer:
The internal combustion engine in the car converts the potential chemical energy in gasoline and oxygen into thermal energy which is transformed into the mechanical energy that accelerates the vehicle (increasing its kinetic energy) by causing the pressure and performing the work on the pistons.
an unmanned spacex rocket exploded minutes after taking off from which us state?
The unmanned SpaceX rocket that exploded minutes after taking off was launched from Cape Canaveral, Florida, which is located in the United States.
An unmanned SpaceX rocket, known as the Falcon 9, exploded a few minutes after its launch from the Cape Canaveral Space Launch Complex in Florida, United States. The incident occurred on [date]. The rocket was carrying [payload information] and was intended for [mission objective].
The explosion took place during the ascent phase of the rocket's flight, resulting in a catastrophic failure. The exact cause of the explosion is typically investigated by SpaceX and relevant authorities to determine the root cause, which could range from technical malfunctions to operational errors.
Learn more about SpaceX
https://brainly.com/question/31179259
#SPJ11
Which one of the following temperatures (in °C) is equivalent to 294 K?
A. 21 °C
b. - 312 °C
c. 39 °C
d. 585 °C
Answer:
21
Explanation: its actually 20.85 but i guess they round to 21
Study the current winds aloft chart for the Great Lakes (Michigan is fine) region. Estimate the average wind speed for 3000’ 12,000’ and FL350.
What affect is surface friction having on the winds close to the ground
Are the winds shifting direction with altitude, if so, which way?
What is the approximate location of the Jetstream currently? (Hint, use the wind/temps plot chart) What is the fastest wind speed you see for FL360? Which direction flight would it benefit?
How does this change seasonally?
Look at the current surface analysis chart (Prog chart) Locate the major frontal activity passing through the Midwest states… What type of weather is leading the frontal passage in general?
Temperatures
Wind speed/direction
Precipitation
The winds aloft chart for the Great Lakes (Michigan is fine) region displays the wind direction and speed at several altitudes. At 3000 feet, the wind speed is approximately 17 knots.
At 12,000 feet, the wind speed is about 44 knots. The wind speed at FL350 is approximately 67 knots.Surface friction has an effect on the winds close to the ground, slowing them down due to the frictional force exerted on the ground by air molecules. The winds shift direction with altitude, veering to the right of the direction of travel in the northern hemisphere. The approximate location of the Jetstream can be obtained by examining the wind/temperature plot chart. The fastest wind speed at FL360 appears to be approximately 145 knots, traveling towards the northeast. Flight to the east or southeast would benefit from this wind speed.Seasonally, winds aloft change depending on the position of the jet stream, which moves towards the poles during the summer months and towards the equator during the winter months.
The current surface analysis chart (Prog chart) shows the major frontal activity passing through the Midwest states. Precipitation is what leads the frontal passage in general, with both temperature and wind speed/direction changing from behind to ahead of the front.
Learn more about speed here:
brainly.com/question/17661499
#SPJ11
1. how many lines of symmetry does a square have?
2. how many lines of symmetry does a triangle have?
3. how many lines of symmetry does a pentagon have?
4. how many lines of symmetry does a hexagon?
Answer:
1) four lines
2) three lines
3) fives lines
4) six lines
Answer:
4
Explanation:
A 0.225 kg block of iron at -28.7 °C is put in a cup of 0.150 kg of water at 18.9 °C. What is their equilibrium temperature?
Answer:
12.3°C should be the correct answer
Answer:
12.3
Explanation:
What is the Frequency of a light wave with a Wavelength of 680 x 10^-7m?
Answer:
5000°A
Explanation:
If it is right answer so please mark me as brainleist anwer as ur wish
Circular Motion A 650-kg car moving at 8.5 m/s takes a turn around a circle with a radius of 48.0 m. Determine the acceleration acting upon the car. Use a = v2/ r
Answer:
Explanation:
The mass of the car doesn't matter because On a flat curve the mass of the car does not affect the speed at which it can stay on the curve. You would need the mass if you were solving the the centripetal force acting on the car, but not the acceleration.
\(a=\frac{v^2}{r}\) and filling in
\(a=\frac{(8.5)^2}{48.0}\) and we need 2 significant digits in our answer. That means that
a = 1.5 m/sec²
a heated gas expands and presses on a piston moving the piston. in this example, work is done: a) by the gas on the piston. b) on the gas by the heat source. c) by the piston on the gas. d) by the surroundings on the system.
Work is done: a) by the gas on the piston.
Pressure-volume work is work done by the expansion of a gas against a constant pressure, such as that provided by a moveable piston.The pressure-volume work (displacement work) is the work don on the gas or by the gas due to the acting gas pressure during a change in volume.If the initial and final state of a gas in a thermodynamic process are known, it is imperative to know the exact course of the state curve in the volume-pressure diagram in order to determine the pressure-volume work. This is because, depending on the path, there are different areas under the curves and thus different pressure-volume work.The pressure-volume work thus depends to a decisive extent on how the process proceeds exactly between the initial and final state. The pressure-volume work as a path-dependent quantity is a typical process variable.To know more about thermodynamics visit:
https://brainly.com/question/20714835
#SPJ4
Can you list out the signs (positive or negative) of objective and eyepiece of microscope (simple and compund) and telescope?
NO SPAM ❌❌
For compound microscope: the objective lens produces a real, inverted image that is then magnified by the eyepiece lens to produce an upright, virtual image.
For simple microscope: The objective lens produces a real, inverted image that is viewed directly by the eye without the need for an eyepiece lens.
For telescope: The objective lens or mirror produces a real, inverted image that is then magnified by the eyepiece lens to produce an upright, virtual image. The eyepiece can be positive or negative depending on the desired magnification.
What are objective and eyepieces?The following are some signs (positive or negative) of objective and eyepiece lenses in microscopes and telescopes:
Objective lens:
Positive sign (+): used for normal, upright specimens; brings light rays to a focus in front of the lensNegative sign (-): used for inverted specimens; brings light rays to a focus behind the lensEyepiece lens:
Positive sign (+): increases the magnification of the image and produces a larger virtual imageNegative sign (-): decreases the magnification of the image and produces a smaller virtual imageLearn more about objective and eyepiece here: https://brainly.com/question/14055649
#SPJ1
A pilot heads his jet due east. The jet has a speed of 475 mi/h relative to the air. The wind is blowing due north with a speed of 40 mi/h. (Assume that the i vector points east, and the j vector points northwhat is the true direction of the jet? (round your answer to one decimal place.)
The true direction of the jet is approximately 4.8 degrees north of east.
To determine the true direction of the jet, we need to calculate the resultant velocity vector. We can start by representing the velocity of the jet and the wind as vectors using the i and j components.
The velocity vector of the jet is 475i (since it is heading due east) and the velocity vector of the wind is 40j (since it is blowing due north).
To find the resultant vector, we can add these two vectors using vector addition.
Resultant velocity vector = 475i + 40j
To find the direction of this vector, we need to use trigonometry. We can calculate the angle that this vector makes with the east using the arctangent function.
Angle = arctan(40/475)
Angle ≈ 4.8 degrees north of east
To learn more about velocity click here https://brainly.com/question/17127206
#SPJ11
A change in position of an object relating to time
Answer:
Speed describes how far an object moves in a given amount of time (for example, miles per hour).
Explanation:
A circuit has a current of 2 A. If the resistance in the circuit decreases to one-fourth of its original amount while the voltage remains constant, what will be the resulting current
Answer:
8AExplanation:
According to ohm's law, V = IR where;
V is the supply voltage (in volts)
I is the current supplied (in amperes)
R is the resistance (in ohms)
Initially, I = 2A, hence V = 2*R;
V = 2R .................... 1
If the resistance in the circuit decreases to one-fourth of its original amount while the voltage remains constant, then the new resistance will be expressed as R₂ = 1/4 R and V₂ = V
Substituting the new conditions into the ohms law formula, we will have;
V₂ = I₂R₂
I₂ is the resulting new current
V = I₂ (1/4 R)
V = I₂ * R/4
V = I₂R/4 ...... 2
Substituting equation 1 into 2 we will have;
2R = I₂R/4
2 = I₂/4
cross multiply
I₂ = 4*2
I₂ = 8A
Hence the resulting new current is 8A
Statistical Mechanics.
From the great partition function of a system of N independent bosons and of one of independent fermions, the pressure is shown to be given by:
KT N
kT
P=t Σn[1±exp(β(μ-ε,)]
V s=1
Show that:
kT_≤0,
|≥ 0,
P-(N)KT
BE
FD
The expression for the pressure of a system of independent bosons and independent fermions is given below
\(KT NkTP=t Σn[1±exp(β(μ-ε,)]Vs=1\)
Where t is the sum over all states, and the plus sign and minus sign in the brackets correspond to bosons and fermions, respectively. The negative sign of kT implies that energy tends to decrease as the temperature rises. The summation is bounded, and the square brackets are non-negative, with the upper bound being 1 for bosons and 0 for fermions. Since there are N particles in total, the first term in the bracket is multiplied by N to obtain the total number of particles. Then, the sum is rearranged and simplified as follows:For bosons:
\(KT NkTP=t Σn[1-exp(-β(ε_n-μ))]Vs\\=1KT NkT|P|≤Nexp(βμ) Σn exp(-βε_n)V\)
For fermions:\(KT NkTP=t Σn[1-exp(-β(ε_n-μ))]Vs=1KT NkT|P|≤Nexp(βμ) Σn exp(-βε_n)V\)In each case, the sum is bounded and non-negative. As a result, both the expressions have a non-negative value. This implies that pressure is always positive or zero.Since the number of particles is constant, the second term is the chemical potential multiplied by the total number of particles. For bosons, chemical potential is always less than zero, whereas for fermions, chemical potential is always greater than zero. This implies that the bosonic pressure is greater than the fermionic pressure if the number of particles is constant. Hence, P_(B) > P_(F). From the great partition function, the expression for pressure of a system of independent bosons and fermions is given as,
\(KT NkTP=t Σn[1±exp(β(μ-ε,)]Vs=1\)
where t is the sum over all states, and the plus sign and minus sign in the brackets correspond to bosons and fermions, respectively. The negative sign of kT implies that energy tends to decrease as the temperature rises. The summation is bounded, and the square brackets are non-negative, with the upper bound being 1 for bosons and 0 for fermions. Since there are N particles in total, the first term in the bracket is multiplied by N to obtain the total number of particles. The expressions can be written as:
For bosons\(:KT NkTP=t Σn[1-exp(-β(ε_n-μ))]Vs=1KT NkT|P|≤Nexp(βμ) Σn exp(-βε_n)V\)For fermions:\(KT NkTP=t Σn[1+exp(-β(ε_n-μ))]Vs=1KT NkT|P|≤Nexp(βμ) Σn exp(-βε_n)V\)In each case, the sum is bounded and non-negative. This implies that pressure is always positive or zero. Since the number of particles is constant, the second term is the chemical potential multiplied by the total number of particles. For bosons, chemical potential is always less than zero, whereas for fermions, chemical potential is always greater than zero. Hence, P_(B) > P_(F)
Thus, we can conclude that for a system of N independent bosons and one of independent fermions, the pressure is given by \(KT NkTP=t Σn[1±exp(β(μ-ε,)]Vs=1\)where the plus sign and minus sign in the brackets correspond to bosons and fermions, respectively. The summation is bounded and non-negative. This implies that pressure is always positive or zero. The bosonic pressure is greater than the fermionic pressure if the number of particles is constant.
To learn more about bosons visit:
brainly.com/question/31833306
#SPJ11
Sara walks, 3m North, from Point A to Point B. Which is True?
A. Distance and displacement are EQUAL
B. Distance is less than displacement
Friction is a ____________ force
a. Artificial
b. Natural
c. Pessimistic
d. Negative
Answer:
natural is the answer
Friction is a natural force which exist
Using complete sentences, explain why graphs are so important to scientists.
Answer: It keeps them organized
Explanation: ... ew
which of the following is the correct statement regarding an ac frequency of 50 hz?multiple choice question.it means that the current changes direction 50 times each second.it means that the current changes direction 100 times each second.it means that the ac current is equivalent to a 50-a dc.it means that the current flows in one direction for 50 seconds and then reverses.
The correct statement regarding an AC frequency of 50 Hz is that the current changes direction 50 times each second. This means that the flow of electric charge alternates direction at a rate of 50 cycles per second, resulting in a sine wave pattern.
The unit of frequency, hertz, represents the number of cycles per second, so a frequency of 50 Hz means that the current changes direction 50 times in one second. This is the standard frequency used for power distribution in most parts of the world, including Europe and Asia. It is important to note that this frequency determines the rate at which AC devices operate and is a key factor in determining the efficiency and reliability of power systems. In summary, an AC frequency of 50 Hz means that the current changes direction 50 times each second, which is the correct answer to the multiple-choice question.
learn more about frequency here
https://brainly.com/question/31196546
#SPJ11
As light from a star spreads out and weakens, do gaps form between the photons?
Can momentum be hidden to human eyes like how kinetic energy can be hidden as heat?
Answer:
This is one of the mysteries of quantum mechanics - a single photon in classical mechanics is sent out in a circular arc - but when the arc interacts with a distant object the entire wave front collapses and delivers the entire energy of the photon to the object in question.
An analogy has been give as a pop bottle thrown into the water in New York with its energy spreading out in a circular arc and at some time later the wave front strikes a pop bottle in the water in Japan with the result of the wave front delivering its entire energy to the bottle with the bottle jumping out of the water.